UP-MBA-GPAT-Reviewer-and-Sample-Questions.pdf

May 12, 2018 | Author: Jayvee Sicat | Category: Percentage, Argument, Fraction (Mathematics), Fallacy, Test (Assessment)


Comments



Description

GRADUATE PROGRAM ADMISSION TESTREVIEWER & TEST QUESTIONS | http://rmnisperos.com GPAT Overview The Graduate Program Admission Test (GPAT) consists of quantitative math, logic, and reading comprehension. The purpose of GPAT is to screen applicants and make sure that only the eligible applicant can be admitted to the program. As no test can measure all aspects of a person’s intelligence, the GPAT measures those skills deemed most critical to a new graduate student. Your GPAT score is one of the most critical elements of your qualification for MBA, so it is naturally much too important for you to take this test unprepared. The higher you’re GPAT score, the better your chances of admission for a respected, competitive graduate program. It won’t take you long to discover that the GPAT is unlike any test you’ve taken before, and it is probably unlike any test you will ever take again in your academic career. The typical high school or college test is a knowledge-based test. The GPAT, however, is skills-based. What does this mean to you? It means that you’ll have to prepare yourself in a completely different way! You won’t simply be reciting memorized facts as they were phrased in some textbook. The GPAT requires you to think in a thorough, quick and strategic manner yet still be accurate, logical and wise. This test is designed to judge your logic, verbal and mathematical ability in the ways that graduate schools feel is vital to the success of first year graduate students. To some extent, you have already gradually obtained these abilities over the length of your academic career. However, what you probably have not yet become familiar with is the capability to use these abilities for the purpose of maximizing performance within the complex and profound environment of a standardized, skills-based examination. There are different strategies, mindsets and perspectives that you will be required to apply throughout the GPAT. You’ll need to be prepared to use your whole brain as far as thinking and assessment is concerned, and you’ll need to do this in a timely manner. This is not something you can learn from taking a course or reading a book, but it is something you can develop through practice and concentration. This guide provides you with the professional instruction you require for understanding the traditional GPAT test. Covered are all aspects of the test and preparation procedures that you will require throughout the process. Upon completion of this guide, you’ll have the confidence and knowledge you need for maximizing your performance on your GPAT. . Weigh the probability of unsure answers. the higher you score is the higher your changes in MBA admission. Often. There is no “passing” score to the GPAT.25 point deduction for each incorrect answer to your final scores Knowing this information before you enter the GPAT examination means that you know exactly what you’re facing that day. Be sure to answer the questions you’re comfortable with. The GPAT Scoring Scale GPAT scoring is not hard to comprehend when it is properly explained. Each correct answer will get you 1 point. However. you will get ¼ or . leaving a question blank is the best option if you really are doubtful to some of your answers. and interpret graphic data. solve quantitative problems. Quantitative Test The Quantitative section of the Graduate Program Admission Test (GPAT) measures basic mathematical skills. and the ability to reason quantitatively. Problem-Solving and Data-Sufficiency questions are intermingled throughout the section. Two types of multiple-choice questions are used in the Quantitative section of the GPAT: Problem Solving and Data Sufficiency. understanding of college concepts. Both types of questions require knowledge of:  complex arithmetic  elementary algebra and calculus  commonly known concepts of geometry Problem-Solving Questions Problem-Solving questions are designed to test:  basic mathematical skills  understanding of mathematical concepts  the ability to reason quantitatively and solve quantitative problems Data-Sufficiency Questions Data-Sufficiency questions are designed to measure your ability to:  analyze a quantitative problem  recognize which information is relevant  determine at what point there is sufficient information to solve a problem .  Use textbooks for increased detail. or. To identify the skills that need extra work. practice! The best way to get to learn your math skills is to rehearse them with as many new sample questions as you can get your hands on. so that you can move on and concentrate on other areas for perfection.Data-Sufficiency questions are accompanied by some initial information and two statements. but statement (1) is not sufficient. complete a practice test and look for yourself at the areas where you excelled. the more you will become familiar and comfortable with that type of question.  Practice. The questions you do. To master your critical math skills. practice. You may answer that: -Statement (1) ALONE is sufficient. assistance. and the areas where struggle was apparent. This is the area in which you can maximize your score increase potential. Those will be the math skills that will best help your score in the shortest period of time. -BOTH statements TOGETHER are sufficient. complete a practice test that gives additional information. You must decide whether the statements given offer enough data to enable you to answer the question. -EACH statement ALONE is sufficient. but statement (2) is not sufficient. there are certain steps you may take:  Read over the skill lesson in this book. -Statements (1) and (2) TOGETHER are NOT sufficient. Your “critical” math skills will be in the areas where you have made the most wrong answers on your practice test. if you manage to practice and better these skills. but NEITHER statement ALONE is sufficient. practicing the new skills that you have learned in through your review. . -Statement (2) ALONE is sufficient. labeled (1) and (2). very carefully  Find some practice tests and work specifically on the questions that test your critical math skills. and question examples for the areas in which you are struggling the most. Since calculators are permitted in the test. Question Types The test is very consistent with the type of mathematics questions that it uses. you’ll need to recall your order of operations.they’re not out to measure your ability with a calculator. the questions will obviously not be purely arithmetic . A good trick to recall your order of operations is “Please Excuse My Dear Aunt Sally”…before you say “huh?” recognize the first letters in this phrase: . multiplication and/or division. subtraction. So in this style of question. The following are the types of mathematical questions that you are likely to encounter: ● Arithmetic ● Divisibility ● Multiplication ● Addition ● Subtraction ● Evens and Odds ● Fractions ● Prime Numbers ● Percent ● Square of a Number ● Exponents ● Roots ● Averages Arithmetic Arithmetic skills refer to the questions that can be solved by using addition. year after year. 28 Note: the answer is 25 Divisibility The factors of integer X are the integers by which X can be divided without leaving a remainder. . 22 D. X is divisible by its factors. 18 C. Often. subtract. 15 B. Thus. multiply and divide. For example: How many egg cartons are needed to hold 300 eggs. if each carton can hold one dozen (1 dozen = 12) A. and will likely test other skills as well. instead of being purely arithmetic.● Work within Parenthesis ● Simplify Exponents ● Multiplication and Division ● Addition and Subtraction The majority of arithmetic questions will require you to take multiple steps. where you will need to decide when to add. 25 E. the questions will be presented in the form of word problems. 2. 6 is divisible by 3 with no remainder). 3. 6. Numbers divisible by 4 can be identified if their last two digits will divide by 4 without a remainder (for example. which is divisible by 9). Numbers divisible by 3 can be determined by adding the sum of their digits and checking if that number is divisible by 3 (for example the number 123: 1+2+3=6. have a look at the following: 1. Numbers divisible by 5 end only in 5 or 0. the number 639: 6+3+9 = 18. which are divisible by 4 with no remainder). 50 E.For example: The number 10 is divisible by both 5 and 2. 5. 7 C. To review the rules of divisibility. 3 B. 10 can be divided by both of these integers without leaving a remainder. A number is only divisible by 10 if it ends in 0 The following is an example of a divisibility question: Which of the following integers divides into both 200 and 150? A. the number 624: the last two digits are 24. Numbers divisible by 2 end in even numbers. 30 D. Numbers divisible by 9 occur when the sum of its their digits are divisible by 9 (for example. 4. 300 Note: The correct answer is (D) . means that X is a positive number X < 0.A (-X)2 = X2 If X .B) = -A + B = B .0.B = A + (-B) A minus B is the same as A plus (the opposite of B) X > 0. Multiplication The following are a few simple rules to keep your multiplications on track: Positive x Positive = Positive Negative x Negative = Positive Negative x Positive = Negative Addition Here are some rules to be certain that there are no slips while doing addition: Positive + Positive = Positive Negative + Negative = Negative Negative + Positive = either positive or negative (you must use the absolute value of both: subtract the smaller from the larger and keep the sign of whichever absolute value was larger) Subtraction The definition of subtraction is: A . means that X is a negative number -(A . X2 > 0 . one number occurs to the left of another number. and Z = 3. For example: Use the number line to make conclusions with regards to whether each number is positive or negative. Y = -2. let X = -7. For example.If. In this situation. -2 . The following is an example of a subtraction question: Y-X Solution: Positive Y is greater than X. when studying the line above. Therefore. you will know that X < Y and Y < Z. Be certain to utilize some negative numbers while substituting.(-7) = -2 + 7 = 5 . on the number line. you will have an easier time if you implement specific numbers to fit the problem. the number on the left is the smallest number. Keep in mind also that 0 is an even number. -2. that an even number is divisible by 2 and not have any remainder. then the next consecutive even number would be represented as X + 2.…}. 3. -4. Properties of odd and even numbers with Addition Property Example Even + Even = Even 2 + 8 = 10 Odd + Odd = Even 3 + 9 = 12 Odd + Even = Odd 3 + 8 = 11 Properties of odd and even numbers with Addition Property Example Even x Even = Even 4 x 6 = 24 Even x Odd = Even 4 x 5 = 20 Odd x Odd = Odd 3 x 9 = 27 Consider the following example: If R is an odd integer. 1. To check the property of a number. on the other hand. you can simply substitute the appropriate numbers. what are the next two consecutive odd integers? . The following charts demonstrate the properties of odd and even numbers. -1. Odd numbers. if x is an even number. Consecutive even numbers are all located 2 units apart. are numbers within the set {…-5. though. 0. Evens and Odds An even number is any word that is divisible by 2: numbers that are within the set {…-6. 2. -3. Remember.…}. 4. 6. 5. For example. 5. and (E) is even. 2. because its factors are 1. 29 . (B) is even. (C) is even. as its only factors are 1 and 7. 17. 3. 6 is not a prime number. 2x + y Note (A) is odd. x + y E. 7 is a prime number. y2 D. However. 19. 6 The first ten prime numbers are: 2. x2 B. 3. and has only two positive factors. 23. 11. A. A) T and V B) R and R+1 C) R+1 and R+2 D) R+2 and R+4 E) R+1 and R+3 Note: the correct answer is (D) Here’s another example: If x is an odd integer and y is an even integer. 7. Prime Numbers A prime number is defined as an integer that is greater than 1. For example. (D) is odd. 13. xy C. tell whether each expression is odd or even. 1 and itself. though that 1 is not a prime number. and both the smallest and the only even prime number is 2. Prime factorization is the process by which you express a number as a result of only prime numbers. 20 E. For example: To create the prime factorization of 24. 13 B. then what is the sum of x + y? A. 16 D. you’d represent it as: 2 x 2 x 2 x 3 or 23 × 3 To create the prime factorization of 15. 26 Note: the answer is B Here is another example: What is the sum of the first 5 prime numbers? A. 28 . 14 C.Note. 18 B. you’d represent it as: 5 x 3 An example of a factor question is: If xy = 13 and both x and y are positive integers. what is the percent of questions she missed? Percent = missed questions x 100 = 12/80 x 100 = 0. C. The answer is B. A percent can be determined by performing the division of the part by the total and multiplying it by 100: Percent = Part x 100 Total For example. 11 and their sum is 28. 5. before multiplying. . 3. 7. while the word “of” means “multiply” However. Converting a number into a percentage involves multiplying the number by 100. 30 D. Percents The word percent means “hundredths” or a number which is divided by 100.15 x 100 = 15% Total The phrase “X is N percent of Y” can also be written mathematically as X = N x Y 100 The word “is” means equal (=). 38 Note: The first five prime numbers are 2. you must change a percent into a decimal or fractional format. 34 E. if Wendy missed 12 out of 80 examination questions. change the fraction 1/5 into the decimal 0.6% or 16.1 10% 1/7 0.0 100% . and then multiply by 100 (or move the decimal point by 2 places to the right) For example: Change the fraction 1/5 into a percent.16666. and may wish to memorize.2 20% 1/4 0.2.20 x 25 To change the fraction into the percent.For example: 5 is 20% of 25. 16. means 5 = 0.7% 1/5 0. you must first change the fraction into a decimal.. Fraction Decimal Percent 1/100 0.25 25% 1/3 0. Therefore: 1/5 x 100 = 20% The following table provides the common percentages that you will use on a regular basis.6 60% 1 1.5 50% 3/5 0. and multiply by 100 (move the decimal 2 places to the right).3% 1/6 0.33 33.1428571 14.. First.3% 1/2 0.01 1% 1/10 0. 20 x 50 Keep in mind that to change the percent to a decimal. 5 B. 10 D. 15 Note: the answer is C To solve this question. 25% . 8 C.5% B.5 150% Please note that numbers over 1 achieve percentages that are greater than 100% Consider the following example: What is 20% of 50? A. 12 E. Therefore. X = 0. 2. 3/2 1. you must rewrite it as an algebraic question. X = 10 Here is another example: 5 is what percent of 2? A. and that the word “of” means that you should multiply. let x represent the unknown number. 250% E. 5 = n × 2 Solve for n and remember to change the answer to a percent. n = 5/2 = 2. n n2 n3(n > 0) n3(n < 0) 1 1 1 -1 2 4 8 -8 3 9 27 -27 4 16 64 -64 5 25 125 -125 6 36 216 -216 . When preparing for the test. the result obtained is called a perfect square. 100% D. 500% Rewrite this as an algebraic equation. the answer is (D) Square of a Number Squaring a number means to multiply that number by itself. make sure that you are fully capable of understanding and reproducing the following table. The notation for squaring a number (x) is as follows: x2 When squaring an integer. as well as recognizing the numbers that are perfect squares and perfect cubes.5 = 250% Therefore. C. 9 D. The following is an example: Of the following numbers. 16 E. or x = -y. 4 B. except for x = 0 ● x2 > x for x > 1 ● x2 < x for 0 < x < 1 *important ● x2 = x for x = 1 or 0 ● The square root of x2 equals the absolute value of x. 64 . then either x = y. which is a both a perfect square and a perfect cube? A. or y = -x. 8 C.7 49 You will not You will not need to know need to know any higher any higher 8 64 9 81 10 100 11 121 12 144 Squared numbers and special properties ● x2 > 0 always. ● If x2 = y2. and x3 is read as x.456 = 3 × 104 + 2 × 103 + 4 × 102 + 5 × 101 + 6 × 100 . followed by the number of zeros as specified by the power. When it comes to the power of 10. The x is the base. An example you may find is: Represent 32. quick rule that simplifies the powers of 10.000. The solution would be as follows: 32. Math questions will usually only utilize integral exponents. All others are read as a power of x. Examples: x3 = x × x × x x5 = x × x × x × x × x The expression of xn is also called the nth power of x. 100000 = 100. while the n is the exponent. x4 is read as the 4th power of x. Examples: 105 = 1 followed by 5 zeros. x2 is read as x-squared. there is a simple. by writing it as 1.456 to the power of 10.Note: the answer is (E) Exponents The mathematical notations for numbers which are the result of a number that is multiplied by itself a number of times is called exponents.cubed. Consider the following example: Solve for x: (x . No radical can be within the denominator. If you try guessing.3)2 = 49.3)2 = 49 x . Just remember x -3 must be positive or negative. do the following: (x . . You could use algebra and take the square root of both sides or since 49 is a perfect square you could guess integers for x. You should remember that none of the following should ever occur: 1. if you cannot "see" the answers fast enough. Roots The test will require you to manipulate both square roots and cube roots. Of course. use other approaches to answer the problem. the integers 10 and -4 work.3 = 7 or x . 2. No perfect square can be left underneath a radical (square root) sign. Some of the questions will measure whether or not you understand these expressions. if you train yourself to use the technique. To get an algebra solution. Sometimes guessing (Guessing in this case means substituting in numbers to see which satisfy the equation.3 = -7 x = 10 or x = -4 It is your goal to get problems correct quickly.) is faster than solving an equation. 35 + 56 + 75 + 28 = 194 194 / 4 = 48. 47 Note: the answer is (B). 75.3. For example: Jenna’s last four test scores were 35. 48. and 28. 54 E. 52. 56. Total 2.5 C. 43 B. Average (also known as a mean) 3.5 D. To discover the average. simply divide the total by the # of numbers.5 Five things to remember when solving averages: . Averages There are three basic components that comprise an average problem: 1. # of numbers The average is the total of elements that are within the set. What is the average of Jenna’s test scores? A. No fractions may occur within the radical sign. 1. the new average will not change. 3. 2. the average will increase. the arithmetic mean is the middle value. If a number is added and it is less than the average. . and they are “balanced” on both sides of the average. 5. If a number is added and it is greater than the average. To discover the average between two evenly spaced numbers. add the first and the last terms and divide them by 2. the average will decrease. If a pair of numbers are added. If a number that is the same as the average is added. 4. economics. and inferential questions. applied. no specific knowledge of the material is required. Failure to do so will mean disqualification. All questions are to be answered on the basis of what is stated or implied in the reading material.). Reading Comprehension Test The Reading and Comprehension section of the Graduate Program Admission Test (GPAT) measures your ability to:  read and comprehend written material  reason and evaluate arguments Two types of multiple-choice questions are used in the Verbal section of the GPAT: Reading Comprehension & Critical Reasoning. Because the Reading Comprehension section of the GPAT includes passages from several different content areas. Reading Comprehension passages are accompanied by interpretive. you may be generally familiar with some of the material. human resource management. Remember: Once you have read the passages. and business-related areas (marketing. You should carefully understand the passages and if possible remember details that are important. etc. the proctor will require you to flip the exam to answer the reading and comprehension question. YOU CANNOT GO BACK TO REVIEW THE PASSAGES. physical or biological sciences. . Reading Comprehension Questions Topics contain material from the social sciences. however. on the basis of that information. reach a general conclusion. Questions of this type involve the interpretation of numerical data or the use of simple arithmetic to reach conclusions about material in a passage. and formulating or evaluating a plan of action.What Is Measured Reading Comprehension questions measure your ability to understand. Questions are based on materials from a variety of sources. This section evaluates your ability to:  Understand words and statements in reading passages.  Understand the logical relationships between significant points and concepts in the reading passages. and apply information and concepts presented in written form. Questions of this type ask you to consider factual statements or information and. Questions of this type test your understanding of and ability to comprehend terms used in the passage and your understanding of the English language. . Questions of this type ask you to determine the strong and weak points of an argument or to evaluate the importance of arguments and ideas in a passage.  Draw inferences from facts and statements in the reading passages. analyze. Critical Reasoning Questions Critical Reasoning questions are designed to test the reasoning skills involved in making arguments. evaluating arguments. No familiarity with the specific subject matter is needed.  Understand and follow the development of quantitative concepts as they are presented in verbal material. What Is Measured This section measures your ability to reason effectively in the areas of:  Argument construction. Reading Comprehension Example: Flying Over the Passage A topic that is hotly debated among test taking circles is whether or not you should read the reading passages before you read the question. Our recommended theory is the flyover. Both theories have their own individual merit and due to the differences in ability and preferences among test takers. properly drawn conclusions. at a bare minimum so that you have a general idea about what the . Questions of this type may ask you to analyze a given argument. or parallels between structurally similar arguments. One theory is that you can save time if you read the questions first and then go back and read over the passage. effectiveness.  Argument evaluation. Questions of this type may ask you to recognize the relative appropriateness. reasoning errors committed in making an argument. factors that would strengthen or weaken a proposed plan of action. well-supported explanatory hypotheses. or assumptions underlying a proposed plan of action. or aspects of the methods by which an argument proceeds. underlying assumptions.  Formulating and evaluating a plan of action. recognize factors that would strengthen or weaken an argument. Questions of this type may ask you to recognize the basic structure of an argument. or efficiency of different plans of action. one method may work better than another for you. Another theory is that you should read the passage first and then go into the questions. You want to spend some time on the passage. Note. Try to sort out the details you picked up on and arrange them into a loose organizational pattern that describes the passage. you don’t want to waste too much time on reading the passage. In part. Remember that your goal in the flyover is not to check it off of a test- taking list of things to do. you should be able to eliminate a number of answer choices that are immediately contrary to your summary. You won’t waste time on the details and yet will have a general idea of what the passage is about and what to expect. You want there to be some purpose behind the flyover and having the definite goal of being able to put together a brief mental summary will allow you to maintain some focus and gain benefit from the flyover – as opposed to just skimming it for the sake of skimming it without actually picking up on anything. You should read it very quickly for a high-level overview (hence the flyover) understanding of what is contained in the passage. Creating a Tentative Summary After you’ve finished your flyover of the passage. As you begin going through the questions and answer choices. if you get good enough at putting together your mental summaries from practice. If . Therefore. you should fly over the passage. Don’t hesitate to reopen an answer choice that you’ve already “eliminated” from consideration and reconsider it as a possibility. however that if you find yourself without any good answer choices remaining (because you’ve eliminated them all) you obviously had to have eliminated the right answer choice. However.questions are going to ask and get your mind into the proper mindset for the series of questions. this is a compromise between the theories that gains most of the benefits of each. take a few seconds and compose a tentative mental summary of what you’ve just read. because much of the detail will be forgotten by the time you get to the questions anyway. Always try to go back and confirm the answer. Just because an answer choice seems right. you’re probably right. Try to remember a vague idea of what the different paragraphs are about. Openings and Endings A main focus of this flyover will be the opening and ending sentences in each paragraph. don’t assume that you overlooked information while reading the passage. Your mind can easily play tricks on you and make you think that you read something or that you overlooked a phrase. so what could possibly be wrong? Does the answer choice actually match the passage. Always try to go back and find the support for the answer choice in the passage.you think an answer choice contradicts your initial summary. For the most part. because this will save you time when answering questions later. Extraneous Information Some answer choices will seem to fit in and answer the question being asked. These are likely to contain the main ideas of the paragraphs and should be mentally tagged for future reference. make sure you never try to just answer the questions from this first flyover. Everything seems to check out. . as your memory will play tricks on you and the writers of the test questions may deliberately have planted a trap for you – remember that they don’t exactly have your best interests at heart. or is it based on extraneous information not even contained in the passage. They might even be factually correct. but are not infallible. Getting into the Author’s Mind A number of questions become much easier when you place yourself into the mind of the author of the passage.when you explain something the way you would if you were talking to your friends and family. you should first focus on the opening and ending sentences of the passage and each individual paragraph. Which words and phrases would you use to explain the principle ideas of the passage? This is called “Kitchen Logic” . So. . then it becomes easier to answer questions that would be easy for the author to answer. always go back to the passage and make sure that the answer choice “checks out. Ask yourself a few different questions: “Why did the author write this passage?” “What was the author trying to say?” What angle is the author taking?” “What is the single most important point the author is trying to make?” Put yourself in the shoes of the author and imagine that you wrote the passage and try to identify what you were trying to describe and how you were trying to describe it.” Using Kitchen Logic When a question asks the test taker to identify a main idea. make it easier on yourself by backing away from the passage and thinking about it in terms of using easy “kitchen logic”. If you take on the opinions and ideas expressed by the author as your own.Unless you are behind on time. when faced with identifying the main idea of a difficult passage. while sitting at your kitchen table. If you can’t find the main idea from these key sentences. then ask yourself how you would describe the passage to someone who had never read it. consent. impartiality D.) indignation. most questions about attitude or emotion could be answered correctly. delightful used. consent Now arrange these in order from negative to positive: ( . if a question asks what sort of attitude an author had towards the passage or subject. eagerness C. fear. Emotional Words Each question will be about a different angle of the passage. gloomy. then look throughout the passage for attitude words that might convey a positive or negative attitude. fear E. disappointing used? A lot of questions could be answered correctly simply by going through and circling all the adjectives in a passage. Another way of handling these situations is to arrange all of the answer choices in a list going from most negative to most positive. For questions asking about the author’s emotions. Are words such as brilliant. Without looking at anything else except for the adjectives in a passage. So. eagerness (+) This will help sort out the different choices and keep you from . impartiality. indignation B. excited. or are words such as depressive. find words in the passage that are adjectives describing emotions. Example: Question: The author’s attitude on this topic is best described as: A. or their synonyms to appear in the passage. These key words will be nouns or verbs in the question or answer choices. Finding the Key Words The strategy of finding certain “give-away” words does not only apply to adjectives in questions about emotions or attitude. Rather than answering based on your memory of the passage. One warning that should be made here is that often question writers may use the exact same word or wording in their answer choices that are used in the passage. To gain that support. you have probably identified the particular part of the passage that will contain the support or justification that you need to correctly answer the question and will allow you to be confident in your answer choice selection. but have done so in such a way as to mislead you. Many questions about specific details will have key words that hold the “key” to finding the right part of the passage to look in for the answer. So. simply because a particular word or phrase appears in an answer choice and also appears exactly the same in a . it definitely is not the most time economical method of finding that part of the passage. A better route is to find key words in the question or answer choices that are likely to stand out in the passage and will enable you to quickly narrow your search down. it follows that you have to identify which part of the passage to look in. Once you find a particular part of the passage that either has the exact key word repeated or a synonym of the key word. then you should scan through the passage quickly looking for either those key words to be repeated in the passage. While reading back over the entire passage may be the most foolproof method of finding that important part of the passage. you always want to have support for your answer choice rooted in a specific part of the passage.overlooking an answer choice and making an easy mistake. Once you’ve identified possible key words. You will need to understand the main idea of the passage in order to make a proper inference about the author’s intent and mindset. You must logically deduce what follows from what the author has stated in the passage. How would you apply the information you have just expressed to a completely different situation? . Anything directly stated by the author is not an inference. You are looking for what can be inferred by the passage. not what is directly stated in the passage. Applying Ideas for Generalizations Generalization questions are similar to inference questions in that you have to go beyond what is directly stated in the passage by the author. Be sure that you reread the answer choice and consider the context that it is in. Making Proper Inferences Questions that ask you to make an inference from the passage will require you to use your own personal judgment. how would you feel about another similar situation? What would either strengthen or weaken your argument. always try to connect the question to the right words in the passage that will allow you to save time in finding the right part of the passage to look in for the answer and will give you the key to the correct answer choice. If you were the author and believed in what you had just written.passage does not make that answer choice correct. In conclusion. It helps to put yourself again in the author’s shoes. to ensure that you are not misled by a cheap trick. The obvious will not be enough to answer an inference question. disturbed C. If you already know the definition of the word. but mentally replace the answer choice you’ve chosen for the word being asked about. Using Context Clues Context clues are a valuable aide in helping you understand difficult phrases or words in the passage. famous D. Therefore. powerful . Example: A passage states: “He was notorious for making decisions on the spur of the moment…” Question: Which of the following words. However. despised E. Once you’ve made your choice of a good definition go back again to the passage and reread that particular section. evil B. if substituted for the word “notorious” would introduce the LEAST change in the meaning of the sentence? A. A number of questions will ask you about the meaning of words as they are used in a given passage. the reason the test writers may have chosen that particular vocabulary word is because it is used in an unusual context. or have some familiarity with it. a common mistake is to go with your first impulse and choose the answer that you immediately recognize. return to the passage and find where the word is used and make sure that you understand how it is being used in the passage. Example: Question: Which of the following best describes the organization of the author’s discussion of this topic? A. Breaking Down Passage Organization In trying to understand the author’s perspective. you will sometimes be asked about how the passage is organized. quiz yourself on each answer choice.” But once you review back over the passage. “famous” fits in better into the context of the sentence of passage. choice C. Many times. How does the author’s main idea get developed and broken down into supporting ideas and statements? As you go through the answer choices for these organization problems. Make sure you can go back and actually find the example in the passage. the simplest way to find the answer is to note how the opening sentence in a passage or paragraph relates to the rest of the passage. . is there an example in the question? Don’t work exclusively from your memory.If you knew that the most common definition for “notorious” meant being known in an unfavorable sense. This gives you: ““He was famous for making decisions on the spur of the moment…. He provides an example – Ask yourself. then you might be tempted to choose choice A. “evil. Read the sentence again and substitute your chosen answer choice for the word it replaces.” which makes sense and is correct. He makes an acknowledgement – Ask yourself. First Word Analysis When asked for main ideas that best summarize the passage. So always read all the answer choices and only choose the one that is the best. is there a comparison in the question? Again. doesn’t make choice A correct. which theory is being discussed? E. remember that it is not enough for them simply to be true. He praises the research – Ask yourself. He discusses a theory – Ask yourself. convince… E. condemn… D. dispute… B. Simply because the author provided an example. where is the acknowledgement made and to whom? D. go back to the passage and actually find the comparison being made and verify that it exists. see if you could make a decision based on those first words alone. not just the first one you read that is factually correct. an easy strategy is to look at the first words in each answer choice and without looking at the rest of the answer choice. The example provided may have been to support a comparison that he was making and the comparison may be the main method of organization. they have to answer the question. He makes a comparison – Ask yourself. C. where is the praise mentioned? After each of these initial questions. describe… C. which in this case would make answer choice B correct. Example: Question: Which of the following best explains the author’s primary purpose? A. B. criticize… . the passage that you will face on the test may almost seem out of context and as though it begins in the middle of a thought process. Understanding the Intimidation The test writers will generally choose passages that will be completely foreign to most test takers. Getting hit by strange reading topics that you don’t recognize. Just remember that the passages themselves will contain all the information necessary to answer the questions and you don’t need any prior knowledge of the topic in order to succeed and do well on the test. D and E. of which you may only have a small part of the original selection. after a particular section is pulled and used for the test passage. and that are dry and boring can be a bit intimidating if you’re not adequately prepared. but don’t plan on that happening. leaving you with “describe” or answer choice B as being correct. etc. You can’t expect the passages to be on a topic with which you have any familiarity. C. Therefore. You won’t have a nice title overhead explaining the general topic being covered but will immediately be thrown into the middle of a strange format that you don’t recognize.If you know that the passage is fairly neutral about the subject. it will likely be dry and boring. If you do happen to come across a passage that you are familiar with. you can probably eliminate the stronger verbs used in answer choices A. . then even if you know nothing else. The passages will also frequently be drawn from longer passages in books. Also. articles. consider yourself lucky. while the topics chosen may have originally been interesting reading in their original state. journals. YOU CANNOT GO BACK TO REVIEW THE PASSAGES. It will take practice to determine what is the optimal rate at which you can read fast and yet absorb and comprehend the information. Remember: Once you have read the passages. You should carefully understand the passages and if possible remember details that are important. Failure to do so will mean disqualification. Train your brain to remember the details and absorb the facts. Experiment with reading articles faster and slower and always gauge how well you comprehended what you read at the end. ask yourself some questions about what you just read and see how well you can comprehend. Read an article at your normal pace and then after you’re finished. the proctor will require you to flip the exam to answer the reading and comprehension question. on the flyover. However. The exam is time-pressured so read as quickly as you can ! . Finding your Optimal Pace Everyone reads at a different rate. you are looking for only a surface level knowledge and are not trying to comprehend the minutia of details that will be contained in the passages. This is true for both the flyover that you should initially conduct and then the subsequent reading you will have to do as you go through and begin answering the questions. You can practice with any form of reading material. you should have a sufficient amount of time to read the different sections of the passages at a comfortable rate. this may be one of the hardest strategies. you will find the pace that you should maintain on the test while going back through passages. not maximum comprehension. This is not a speed reading exercise. and yet one of the most important. you want to optimize how much you comprehend with how much time you spend reading. where you are panicked and are buzzing back and forth through the passage in a frenzy and not comprehending anything. and don’t spend too much time on any question. Don’t be a Perfectionist If you’re a perfectionist. allowing you to have time for every question and give you optimal comprehension. Note that you are looking for optimal comprehension. and the overwhelmed mode. you would have maximum comprehension. It should be a comfortable rate. . The test you are taking is timed. Practice will allow you to determine that optimal rate. If you have a good pace.With practice. in which you are lip reading every word individually and mouthing each word as though in a stupor. If you spent hours on each word and memorized the passage. That isn’t the goal though. and you cannot afford to spend too much time on any one question. You must find your own pace that is relaxed and focused. The two extremes you want to avoid are the dumbfounded mode. Before you mark it as your answer choice. you might go . along with the viewpoint of other experts or other individuals. You feel that if you just spent one more minute on the problem. that you would be able to figure the right answer out and decide between the two. and so are actually wrong. Therefore. first make sure that you go back to the question and confirm that the answer choice answers the question being asked. get off track and end up spending the rest of the test playing catch up because of all the wasted time. but fail to answer the question. This can lead to trouble in answering questions though. Different Viewpoints Some passages will express the author’s viewpoint on a topic. Factually Correct. It is far better to accept that you will have to guess on some questions and possibly get them wrong and still have time for every question. which may leave you rattled and cause you to miss even more questions that you would have otherwise. When you are going through the answer choices and one jumps out for being factually correct.If you are working on a problem and you’ve got your answer split between two possible answer choices. but Actually Wrong A favorite ploy of question writers is to write answer choices that are factually correct on their own. you can be in one of the most frustrating situations possible. you should not go into the test with the mindset that you’ve got to get every question right. watch out. than to work on every problem until you’re absolutely confident in your answer and then run out of time on the last few problems.1% strata of test takers. and you’re going back through the passage and reading it over and over again in order to decide between the two. unless you will only be satisfied with a perfect score and your abilities are in the top . If asked for the viewpoint of the author. Watch out! You can easily get so absorbed in that problem that you loose track of time. Make sure that if multiple individuals are giving their viewpoint on a topic.back to the passage. but it is extremely dense and will require very careful reading. but when other viewpoints besides the author’s are expressed. that you sort them out for any questions and associate the right viewpoint with the right individual. where you must identify the flaw. Most commonly. It’s your job to dissect these arguments to find the assumptions that have been made. the questions will involve flawed arguments. you will need to locate the conclusion of the valid argument. answer the question based on what you read and move on. as they will train you to identify . which will fall into one of ten error types (including ad hominem or fallacy of equivocation). you’ll experience deductive arguments that are closer to logic games. find where a certain viewpoint is expressed. such as the ad hominem fallacy . The text may be substantially shorter than the Reading Comprehension section. and again sometimes. Other times. that would be fine. Critical Reasoning The questions of the Critical Reasoning section typically present an entire argument within the span of a few sentences.the attack of an opponent as opposed to the argument itself). Practice GPAT tests will give you an extremely substantial advantage when it comes to this portion of the test. Most questions contain some sort of error. you have to discern who is expressing their opinion in the passage. The majority of the Critical Reasoning passages are between 3 and 4 sentences and will apply to a single question. Next you will need to locate and identify the different ways in which the questions have been phrased. For most passages. It is the flaws within the arguments that are the logical fallacies. an argument is defined as a presentation of facts and opinions with the purpose of supporting a position. is that only two of the five choices presented to you will have any real merit. Hence. if you over-concern yourself by trying to find the truth. only one attractive wrong answer choice will normally be presented to you. So even if you don’t completely understand the argument. you should be able to eliminate the 3 fluff choices. which dramatically increases your odds of selecting the correct answer.” this little trick is called the “Two Out of Five Rule. When you create a good. and not the truth of the statements themselves. is that you can not only comprehend the arguments. Next you’ll need to work on your plan of attack.” What it means to you. it can be detrimental to your GPAT score. ❒ Finding out what the question is asking. though inaccurate answer-choice is much more challenging than producing the correct answer. . Logic. The theory behind the GPAT Critical Reasoning section questions is designed to be answered without any reference to actual formal logic. ❒ Choosing the correct answer from the list of choices. What the GPAT wants to prove. a process that allows you to work on each question by: ❒ Identifying the logical error. Therefore. but can also locate any fallacies that may occur within the argument. As far as the GPATs are concerned. and then properly understand the question that is being posed regarding this error in logic. according to the GPAT.the logical error in the question. is the study of the relationship that occurs among statements. therefore testing your ability to think logically. To those “in the know. However. this is not to say that some fundamentals of logic will not give you an advantage. though rarely will you find it in the middle. Occasionally. the conclusions of an argument are located at the very end. Identifying Premises and Conclusions The majority of argument questions are centered . If you’re uncertain as to where the argument is specifically located. you can look for certain words that writers utilize for indication that the conclusion is about to occur. The conclusion is defined by the GPAT as the main purpose and idea of the argument.therefore it is a true test of logic and not an understanding of logic as a study. Often. The conclusion is what the writer of the argument is attempting to persuade the reader into believing. the conclusion will be located closer to the beginning. however.either directly or indirectly . this is because writers aim to organize facts and opinions so they build-up to a climax for the arguments ending.on establishing the conclusion of the argument itself. These conclusion indicators include: ❒ Hence ❒ So ❒ Thus ❒ Follows that ❒ Conclude that ❒ As a result ❒ Therefore . because an understanding of logic basics will provide you with definite benefits. these are called quantifiers and should be carefully watched for. Frequently GPAT authors will create wrong answer choices by forming a slight overstatement or understatement of the passage author’s conclusion. The following is a list of the most common and important quantifiers: ❒ All ❒ Some ❒ Only ❒ Never ❒ Probably ❒ Except ❒ Most ❒ Could ❒ Always ❒ Must ❒ Likely ❒ Many ❒ No ❒ Everywhere . Several words are employed to limit the scope of a statement.❒ Accordingly ❒ Consequently ❒ Shows that ❒ Implies ❒ Means As you determine the scope of the conclusion. take care not to read further into the conclusion than the author has stated and intended. To discover whether or not a particular statement within an argument is a premise. some premises are left unstated because there are too many of them. you must simply ask yourself if it supports the conclusion. These premise indicators are: ❒ Because ❒ Is evident that ❒ Since ❒ In that ❒ If ❒ Owing to ❒ As ❒ In as much as ❒ Suppose ❒ May be derived from ❒ Assume Most commonly. More often. There are many words used by writers that should be considered to be flags of premises.” The premise is the portion that offers evidence for the conclusion of the argument. as it may be an oversight made by the writer. or that the writer wishes for the . an argument is dependent upon either one or more than one premise which is unstated.❒ Alone Once the conclusion has been determined. almost everything else in the argument will comprise either of premises or “noise. and the writer is assuming that the reader is aware of these assumptions. This will frequently indicate the weakness of an argument. The premise is the foundation upon which the conclusion is built. though. audience to include their own premise. while presenting a position within an argument. Of course. like premises also have word indicators which will help you in their location. then it is likely to be the underlying premise. It also works to disarm potential arguments that might be made against the argument’s position. simply ask yourself whether or not the answer-choice makes the argument more believable. however. If it does. This type of statement is called a counter-premise. To test whether or not a possible answer-choice is the correct one. These suppressed premises within an argument are often sought out within questions on the GPAT. These words include: ❒ But ❒ Admittedly ❒ Even though ❒ Nevertheless ❒ However ❒ Despite . it is often an accepted technique for efficacy to concede certain negligible points that weaken the argument. The location of these tacit premises and assumptions can be quite challenging. though you will have an advantage with the GPAT. the writer will not wish to present a counter-argument to their own conclusion. so that they will be more likely to believe and/or agree with the conclusion. as you will know that the implicit premise is listed for you as one of the five choices of answer. and that the writer’s ideas have been well considered. This functions to demonstrate the open-mindedness of the writer. and it. then statement. as you have likely guessed. After all. The following is an explanation of the principles of an if-then statement: The premise of an if-then statement is that if the statement is true. Almost every argument is based on one variation or another of an if. However. These wily answer-choices are frequently quite tempting. or is it merely a slight matter? Diagramming Conditional Statements In reality. defining . This is the typical.❒ Except ❒ Nonetheless ❒ Although ❒ In spite of the fact Of course. Instead. and they are partially true. it is extremely rare that you will ever be required to actually draw a diagram. is rather a misnomer. However. as it may be embedded within other similar structures. the term “diagramming” itself. you should ask yourself whether or not it is the main point that the author is trying to make. you will represent the arguments symbolically. the if-then is not always obvious. as they refer directly to a passage within the text. the writers of the GPAT passages and questions will frequently use counter-premises for the purpose of baiting wrong answer choices for selection. then the conclusion must also be true. when selecting your answer. and therefore a poor score will be achieved. a letter represents one element of the argument. it works as follows. or simply a phrase or a clause.” and the obligatory conclusion is “B.” and “a poor score will be achieved. This could be an entire sentence. When symbolizing an argument. Ashley will not study for the GPAT.” where the affirmed premise is “A.” By substituting these symbols for the statement of argument. but simply laying it out in this fashion can be extremely helpful in demonstrating the logical structure within a given argument. B To explain this diagram. you may illustrate it in the form of a “diagram” like this: A€B A Therefore. the following diagram will occur: C€D C . For the preceding example.feature of a conditional statement. Please consider this example: If Ashley doesn’t study for the GPAT examination.” can be symbolized with a “D. then a good score will not be achieved. The if-then statement itself is contained in “A€B. the clause “Ashley will not study for the GPAT” can be represented with “C.” This may seem very simple. as the if-then thought is frequently embedded within other structures of the argument. but essentially. then she will not be accepted to a business school. the if-then statement is not always obvious.then structure. Be very careful with this type of embedded statement. This will require you to learn to spot these structures. is that the argument includes a valid if. At first glance. However. “if Amy is accepted to business school. This is an extremely common mistake among students. The conditional statement is illustrated with “C€D”. It only says that if she does not do well on the GPAT. she will be accepted to business school. then she must have done well on her GPAT. Here is an example of an embedded if-then statement: Amy will be accepted to business school only if she does well on her GPAT. D What this demonstrates. and the obligatory conclusion D then stated. take your re-written argument. this statement does not appear to contain a standard if-then statement. which demonstrates the if-then statement.” The clause “if .” The original statement does not make such a guarantee. it says that if Amy is accepted to business school then she must have done well on her GPAT. This statement does not mean that “if Amy does well on her GPAT. To create the diagram. with its premise affirmed as C.Therefore. then A is also false (contra-positive) If. if we know that the premise is false. Here you will learn to classify and understand the major types of inductive arguments.in any other case. it is an inductive argument. Classification The bulk of the arguments that you will see on the GPAT will be inductive. the if-then statement says nothing with regards to the conclusion. the if-then says nothing with regards to the premise. however. Moreover. then B is also true 2. F Keep in mind that an if-then statement will tell you only two things: 1. If B is false. Therefore. If A is true.Amy is accepted to business school. The validity of an . Within an inductive argument. its writer presents the premises as evidence or reasons for the accuracy of the conclusion. An argument can only be deductive if its conclusion follows from its premises . we know that the conclusion is true.” Therefore: E€F E Therefore.” can be replaced with the term “E. the understanding of inductive statements is a natural part of your preparation process.” and the clause “then she must have done well on her GPAT” can be substituted with “F. inductive argument’s conclusion depends wholly on how convincing the premises are. Unlike deductive arguments, where the conclusion is relatively easy to locate, an inductive argument’s conclusion is never quite certain. In fact, the truth of the conclusion of an inductive argument can range from being highly likely to highly unlikely. Naturally, within a reasonable argument, the conclusion is likely. However, within fallacious arguments, the conclusion is bound to be improbable. To be prepared for the GPAT, you will need to be able to recognize both reasonable and fallacious arguments. To begin, there are 3 major types of inductive reasoning: 1. Generalization 2. Analogy 3. Casual Each of these types of inductive reasoning has their own strengths, though they also have associated fallacies. Hence, it is beneficial to you to be able to classify the type of reasoning within an inductive argument, to more easily allow you to recognize the type of fallacy you may find within. Generalization and analogy are the primary tools with which we amass knowledge and analyze this knowledge within our world. In fact, generalization itself is often referred to as “inductive reasoning,” as it associated with the term “to generalize.” This, unfortunately, carries a negative connotation in terms of effective argument, though in reality, argument by generalization is neither inherently good nor is it bad. The validity of a generalization is dependent upon the context of its argument, and the likelihood of the accuracy of the conclusion; and the less comprehensive a conclusion is, the more likely it is to be a valid conclusion. Please consider the following example: During the late seventies when Japan was rapidly expanding its share of the American auto market, GM surveyed owners of GM cars and asked them whether they would be more willing to buy a large, powerful car or a small, economical car. Seventy percent of those who responded said that they would prefer a large car. On the basis of this survey, GM decided to continue building large cars. Yet during the '80s, GM lost even more of the market to the Japanese. Which one of the following, if it were determined to be true, would best explain this discrepancy? A) Only 10 percent of those who were polled replied. B) Ford which conducted a similar survey with similar results continued to build large cars and also lost more of their market to the Japanese. C) The surveyed owners who preferred big cars also preferred big homes. D) GM determined that it would be more profitable to make big cars. E) Eighty percent of the owners who wanted big cars and only 40 percent of the owners who wanted small cars replied to the survey. The argument makes a generalization from the survey with regards to the general car-buying population; hence, the reliability of the prediction is dependent upon the representativeness of the sample. At first, the choice (A) seems like the obvious answer, as 10% does seem like a large enough amount. However, political opinion polls are, on average, based only upon 0.001% of the population. Even more important is the fact that we don’t know what percentage of GM vehicle owners actually received the survey. Option (B) states simply that Ford has erred in the same way as GM. Choice (C) is entirely irrelevant. Option (D) doesn’t explain the discrepancy, but rather provides even more of a reason for GM to continue its production of large cars Selection (E) demonstrates that part of the survey did not represent the entire public. Therefore, the correct answer is (E). so too may life in the fast lane lead to an early death. so too may dwelling on the negative cause depression. C) Just as a plant can be killed by over watering it. the less ambitions the conclusion is. E) Just as an actor may become stressed before a performance. Additionally. stating that as two things are similar in some respects. so too do one's career opportunities come unexpectedly. Which one of the following most closely parallels the reasoning used in the argument above? A) Just as the bow may be drawn too taut. The argument will be weakened. Consider the following example: Just as the fishing line becomes too taut. so too the trials and tribulations of life in the city can become so stressful that one's mind can snap. if dissimilarities are illustrated. where a claim is made. so too can drinking too much water lead to lethargy. however. the stronger the argument. . D) Just as the engine may race too quickly. so too may one's life be wasted pursuing self-gratification.The next classification of inductive reasoning is analogy. The nature of this classification makes it so that the greater the similarity between the two factors of comparison. B) Just as a gambler's fortunes change unpredictably. the stronger the argument. they will also be similar in other respects. The argument is making a comparison of the tautness of fishing lines with the stress in city life. The conclusion states that the mind can snap in a similar way to a fishing line. Thus, you are seeking an answer-choice that makes a comparison between these two things, while drawing a conclusion based on their similarity. You will look for an argument that utilizes similar reasoning, though not necessarily concepts that are similar. In fact, you should probably be suspicious of any answer- choice that utilizes the words tautness or stress, as they will likely be baited same- language traps. The choice (A) utilizes the same-language, baited trap “too taut.” Additionally, the analogy between the taut bow and self-gratification is a weak one, if even existent at all. Option (B) presents a good analogy, however it hasn’t a conclusion. Answer-choice (C) proffers both a good analogy and a good conclusion; although the conclusion “leads to lethargy,” is an understatement of the scope of the analogy’s implication. Selection (D) gives a strong analogy and a strong conclusion. The same scope as the original is also utilized as: “The engine blows, the person dies,” similar to “The line snaps, the mind snaps.” This is a very probable selection for the best answer, though you should still examine every answer choice to see if a better option presents itself. Option (E) uses language from the original text, “stressful,” making it an obvious candidate as a same-language, baited trap. This trap has been used to make the weak analogy more tempting. Thus, the best answer is (D). Of all three classifications of inductive reasoning, causal reasoning is the most prone to fallacy and is the weakest overall. Nevertheless, it is still a common method of thought. Arguments of causation make claims that one thing causes another. A causal argument does have the potential to be a strong one, but it all depends on the context. There are two primary fallacies that are associated with causal reasoning. ● The confusing of correlation with causation - This fallacy can be represented by stating that A caused B, because A occurred immediately before B. This is clearly a questionable line of reasoning, as the fact that A and B occurred together could have been simple coincidence. ● The confusing of necessary conditions with sufficient conditions - This fallacy can be demonstrated with the belief that as A is sufficient for B, then B cannot occur without A. In fact, while A does cause B to occur, B can still occur without A. Seven Common Fallacies There are seven fallacies that are common to all three classifications of inductive argument. These common fallacies are: ● Contradiction ● Equivocation ● Circular reasoning ● Shifting the burden of proof ● Unwarranted assumptions ● Appeal to authority ● Personal attack A contradiction occurs when two opposing statements have been asserted simultaneously. However, the arguer will typically attempt to obscure the point of contradictions to make the argument more compelling. Equivocation takes place when the writer of the argument utilizes a single word in more than one sense within the same argument. This is a common technique among public speakers and politicians who are seeking to maintain an “out” for themselves. This means that should someone object to a particular statement, the arguer is simply able to change the claim to the other meaning that is not under question. Circular reasoning means that the arguer utilizes assumption as a premise for the conclusion that is being made. Intuitively, it appears as though no one would fall for an argument of this nature; however, the conclusions within circular reasoning will frequently make an additional statement, or an argument may be so entirely long and drawn out that the reader of the argument can forget that the conclusion was stated as premise. Shifting the burden of proof is a natural instinct for a writer who struggles with proof. It is, however, the responsibility of the arguer to provide the evidence in support of the position of the argument. This makes the implication that a particular position is true simply because it has not been disproved by anyone else. Unwarranted assumptions are the fallacy committed when an argument’s conclusion is based on either an implicit or explicit premise that is false or unwarranted. An assumption is unwarranted when it is a false assumption – these premises are usually suppressed or written in a vague way so that they do not receive as much notice or weight. Assumptions are also unwarranted when they are true and yet do not apply within the context of the argument. These are usually explicit premises. An appeal to authority occurs when an expert’s opinion has been cited as support for the arguer’s opinion. Though this method may not necessarily be fallacious, it does risk being so for a number of reasons. Obviously, the though it is wise not to overeat. If you’ve got butterflies in your stomach. neck and shoulders. Getting Ready For Test Day You’re all set to take your GPAT! Now here are a few things to remember for test day: Get there early. if you misread direction. plus it’s distracting to listen to your stomach growl. also know as ad hominem is simply a challenge of the person’s character instead of the opinions being stated. or think illogically due to rushing. Remember. A personal attack. and you’ve had a good night’s sleep. haste makes waste! . Remember. accidentally fill in the wrong answer-choice. feed them! You’ve already done all the practice tests you can do. And most importantly. you don’t want to rush. healthy breakfast . Although there are time restrictions. Place your hand over your heart while taking a very slow. Stay on track. Pay attention to traffic reports so that you can compensate for any unexpected issues on the road. Know exactly where the test will be held and how you will get yourself there. you’ll have time to use the rest room. Your body and mind will need the energy. Now it’s time to get a good. it won’t be worth all the time you save. you only want to perform in a timely manner. and you won’t be pulled over by the first officer who has to fill his speeding ticket quota. Leaving early will mean that you’ll be more relaxed. and whether this person is an expert in the relevant field of the argument.reasonableness of the argument that cites an authority is dependent upon the expertise of that particular authority. deep breath. red traffic lights won’t raise your stress level. Give yourself a massage! Rub your head. so be careful at your guesses as any answers that you are unsure of could result in . or dawdle over questions that you’re struggling with. catch a movie with some friends and relax! . remember that once it’s over.Also. skip that question and just go back at it if you still have time. keep in mind that incorrect answers would count against you. because you did your best.25 point deduction. Most importantly (at least to your sanity). it’s over. Clear your mind of it. Moving through a test methodically and efficiently will likely mean that you’ll have more time at the end than if you were to rush and stumble. If you feel the question is somehow difficult. Go treat yourself. SAMPLE QUESTIONS . 52% 3.5% D. Jack and Kevin play in a basketball game. The average of six numbers is 4. Factor the following expression: x2 + x – 12 A. which of the following could NOT be the total number of points scored by the two boys? A. 40% C. 34 is what percent of 80? A. -20 E. What is the next-highest prime number after 67? A. what is the value of a2 + 3ab – b2? A. 28 E. If a = 3 and b = -2. 44. 68 . 9 6. what is the average of the other four numbers? A. 14 C. (x + 6) (x – 2) D. (x + 4) (x – 3) 5. 6 C. 16 D. 13 2.5% E. 34% B. Problem-solving Solve these problems and indicate the best of the answer choices given. (x – 4) (x + 4) B. All numbers used are real numbers. 1. 7 D. (x – 2) (x + 6) C. 42. If the average of two of those numbers is 2. -4 D. 35 4. (x – 4) (x + 3) E. 5 B. 7 B. If the ratio of points scored by Jack to points scored by Kevin is 4 to 3. -13 C. 8 E. 5 B. Sheila. and Karen. Dean’s Department Store reduces the price of a $30 shirt by 20%. $26. 76 7. Janice. 64 E. 1/3 D. $28. 0. how much of the job could Janice and Karen do in one day? A. 0. $24. 1/9 E. 8 C. 73 E. 1/8 11. 54 D. 1/5 B. how many newspapers can he deliver in 2 hours? 12. Dave can deliver four newspapers every minute. Solve: 0. 0. 75 B. How many 3-inch segments can a 4.60 9. What is the final price of the shirt? A.80 E. 2 B.5 8. 45 C. 1/4 C.075 C. 84 10.03 = A. At this rate.75 D. 32 E. $30 D. 7. Working at the same rate. B. 71 D. working together at the same rate. can complete a job in 3 1/3 days. $32 C. 15 B. 69 C. but later raises it again by 20% of the sale price. 46 28 A. 64 . 16 D.5-yard line be divided into? A.40 B.25 x 0.0075 E. 76% E. labeled (1) and (2). and c = 1. 1/6 E. What is 20% of . If a = 4. b = 3. 12 C. it will be ½ full. 72% D. 84% 15. 16 E. 1/4 D. 20 D. A. Archie’s gas tank is 1/3 full. If Archie adds 3 gallons of gas to the tank. 18 Data sufficiency This Data Sufficiency problem consists of a question and two statements. You have to decide whether the data given in the statements are sufficient for answering the question. 65% C. 28 B. 48% B. 16. (2) Jonathan makes less money than Alice. What is the capacity in gallons of Archie’s tank? A. then A. 1/3 C. 2/7 12 14. in which certain data are given. using only the data given in the statements and your knowledge of mathematics and everyday facts (such as the number of days in July or the meaning of counterclockwise). . Does Jonathan get paid more than Deborah? (1) Alice gets paid more than Deborah.13. expressed as a percentage? 5 A. Statement (1) ALONE is sufficient. a(b c) b(a b c) 4/13 B. but statement (2) is not sufficient. EACH statement ALONE is sufficient. but statement (2) is not sufficient. Statement (1) ALONE is sufficient. B. E. Statement (2) ALONE is sufficient. C. EACH statement ALONE is sufficient. Statement (1) ALONE is sufficient. BOTH statements TOGETHER are sufficient. What is the value of the integer P? (1) P is an integer multiple of 2. Statement (2) ALONE is sufficient. Is a an integer? (1) a > 0 (2) 42 + 32 = a2 A. BOTH statements TOGETHER are sufficient. Statement (1) ALONE is sufficient. but statement (2) is not sufficient. B. E. E. 19. Is the perimeter of a given rectangle greater than 8 inches? (1) The two shorter sides of the rectangle are 2 inches long. but statement (1) is not sufficient. but statement (1) is not sufficient. C. C. but NEITHER statement ALONE is sufficient. D. Statement (1) ALONE is sufficient. EACH statement ALONE is sufficient. 17. D. B. but NEITHER statement ALONE is sufficient. . Statements (1) and (2) TOGETHER are NOT sufficient. Statement (2) ALONE is sufficient. Is the integer a less than the integer b? (1) a3 < b3 (2) a2 < b2 A. Statement (2) ALONE is sufficient. but statement (1) is not sufficient. 18. BOTH statements TOGETHER are sufficient. but statement (2) is not sufficient. D. EACH statement ALONE is sufficient. Statement (2) ALONE is sufficient. but NEITHER statement ALONE is sufficient. Statements (1) and (2) TOGETHER are NOT sufficient. but statement (2) is not sufficient. BOTH statements TOGETHER are sufficient. B. but NEITHER statement ALONE is sufficient. and 5. but statement (1) is not sufficient. E. B. 4. but statement (1) is not sufficient. Statements (1) and (2) TOGETHER are NOT sufficient. 20. Statements (1) and (2) TOGETHER are NOT sufficient. (2) The length of the rectangle is 2 inches greater than the width of the A. (2) 40 < P < 70 A. D. C. Dennis. B. 21. Statements (1) and (2) TOGETHER are NOT sufficient. A. BOTH statements TOGETHER are sufficient. Brian is dividing 50 marbles into 3 groups. 23. Carl. BOTH statements TOGETHER are sufficient. D. Statement (1) ALONE is sufficient. EACH statement ALONE is sufficient. D. B. How many marbles are in the largest of the three groups? (1) The sum of the two smaller groups of marbles is equal to the largest group of marbles. B. C. Is x greater than y? (1) x > 2y (2) x – y > 0 A. BOTH statements TOGETHER are sufficient. Statement (1) ALONE is sufficient. Statement (1) ALONE is sufficient. but NEITHER statement ALONE is sufficient. but statement (1) is not sufficient. (2) The average of the test scores of Angela and Dennis is 84. but statement (2) is not sufficient. E. C. but statement (1) is not sufficient. but statement (2) is not sufficient. B. Statement (2) ALONE is sufficient. C. Statement (2) ALONE is sufficient. A. C. Statement (1) ALONE is sufficient. but NEITHER statement ALONE is sufficient. Barry. Statements (1) and (2) TOGETHER are NOT sufficient. D. but statement (1) is not sufficient. but statement (2) is not sufficient. but NEITHER statement ALONE is sufficient. EACH statement ALONE is sufficient. E. but NEITHER statement ALONE is sufficient.452(b) > 0 (2) –b < 0 A. E. Is b a positive number? (1) 1. EACH statement ALONE is sufficient. What is the average test score of Angela. BOTH statements TOGETHER are sufficient. Carl. and Edward is 87. 22. but statement (1) is not sufficient. E. 24. EACH statement ALONE is sufficient. and Edward? (1) The average of the test scores of Barry. Statements (1) and (2) TOGETHER are NOT sufficient. but statement (2) is not sufficient. Statement (2) ALONE is sufficient. Statement (2) ALONE is sufficient. (2) The smallest group contains 6 marbles. D. Statements (1) and (2) TOGETHER are NOT sufficient. . select the best of the answer choices given. Consequently. but statement (2) is not sufficient. Most federal prisoners receive a high school diploma while incarcerated. READING AND COMPREHENSION For these questions. E. . but NEITHER statement ALONE is sufficient. EACH statement ALONE is sufficient. D. E. On the other hand. BOTH statements TOGETHER are sufficient. EACH statement ALONE is sufficient. 27. C. Statement (2) ALONE is sufficient. 25. The general public does not pay attention to movie reviews. welfare. B. BOTH statements TOGETHER are sufficient. Movie reviewers exert influence on the movie quality. The latest movie by a certain director gets bad reviews before it opens in theatres. E. The most important determinant of success in life is education. which is also unsuccessful. or divorce. A recent study demonstrated a link between education and lifetime earnings. B. but statement (1) is not sufficient. Even children from broken or dysfunctional homes tend to establish themselves as solid citizens so long as they obtain a high school education. C. C. The movie reviewers were right about the first movie. is it an odd number? (1) y3 ≥ 0 (2) y is either an odd number or a negative number A. This director makes terrible movies. very few people go to the movie and the director is given much less money to make his next movie. D. D. Statements (1) and (2) TOGETHER are NOT sufficient. children who fail to earn a high school diploma are much less likely to avoid prison. B. Statement (1) ALONE is sufficient. Statements (1) and (2) TOGETHER are NOT sufficient. but NEITHER statement ALONE is sufficient. What can be inferred from this scenario? A. Which of the following statements most effectively strengthens the above argument? A. 26. If y is an integer. The director will not make another movie. All A are C. Some A are C. No A are C. E. only 28% of individuals between the ages of 18 and 25 voted. 31. None of the above. Shakespeare is the greatest writer of all time. a referendum on lowering the legal age for purchasing alcohol to 18 was on the ballot. Which of the following facts most significantly weakens the above argument? A. Turnout among African-Americans between 18 and 25 decreased from 2000 to 2004. and are therefore not beholden to any particular group. This argument fails to make a clear claim. In the 2000 local election. This argument introduces irrelevant evidence. E. The turnout among voters between the ages of 35 and 44 was 42% in 2004. 72. Research indicates that college graduates from abusive homes are more likely to be arrested. This argument disproves its own premise. C. members of the school board receive no payment. E. C. and 64. D. No C are A. B. however. Which of the following statements most effectively challenges the reasoning above? A. The polls stayed open later on Election Day in 2000. Which of the following is true? A. This argument uses ambiguous language. This is because he wrote the greatest plays. D. so turnout was slightly higher at 39%. (1) All A are B. 30. In 2004. and the greatest writer is the one who composes the greatest works. 29. In the 2004 local election. C. Individuals with heart problems are more likely to have postgraduate education. Children from functional homes are more likely to attend preschool. . After all. 28. D. Which of the following pieces of information weakens the above argument? A. E. candidates made more of an effort to appeal to these younger voters. (2) Some B are C. Members of Congress can also serve on the school board. B. C. B. D. The candidates for city council were ages 55. Members of Congress should not be paid. This argument assumes what it claims to prove. Members of Congress typically have been successful in their prior professional lives. which must get better gas mileage. Jacobson would probably welcome a new shopping center near his practice. He fails to recognize that Dr. B. In their game against the Wildcats. Jacobson’s argument. No brands of natural peanut butter contain preservatives. The Wildcats are the best football team in the league. Why is the mayor’s argument weak? A. He does not challenge Dr. C. while school board members have time to pursue other occupations. Jacobson stood up at the recent town hall meeting and declared that building a new shopping center at the corner of George and Vidalia Streets would be a bad move. they scored 24 points. Which of the following statements would logically complete the argument with the above premises? A. All German cars are safe. 34. Jacobson’s practice. but merely challenges him personally. Lopez is a woman. The peanut butter in Dave’s cabinet contains preservatives. and would be very dangerous for motorists and pedestrians alike. Dr. Members of Congress are not allowed to show favoritism to any particular group. Congress only is in session during part of the year. The Tigers lost to the Wildcats. D. D. A few of the candidates for governor are women. Dale drives a German car. The hammerhead is a kind of shark and therefore has a tailfin. He cited transportation department statistics indicating that the intersection would become overloaded with traffic. Some days of the week are Saturdays and Sundays. The mayor dismissed Dr. B. C. Jacobson’s opinion. E. The Tigers are not a very good football team. so his car is safe. B. He does not support his view with statistics from the . C. Dr. B. D. He does not acknowledge the location of his own office. The Tigers probably lost to the Wildcats. The Wildcats probably lost to the Tigers. Being in Congress is a full-time job. Every shark has a tailfin. 33. The newest cars often get better gas mileage. Today is neither Saturday nor Sunday. Helen has a new car. E. on the grounds that the proposed shopping center is within a block of Dr. D. C. 32. E. The Tigers football team usually loses when they score fewer than 30 points. Which of the following arguments contains logic that closely resembles that of the preceding argument? A. Franklin truly says. Earning money is much less difficult than managing it properly. Which of the following statements best expresses the main idea of the passage? A. "we understand this: this is economy. descriptive B. He doesn’t realize that Dr. Franklin advocated getting a job in a mill. What would this author’s attitude likely be to a person unable to find employment? A. B. Mr. Getting a job is easier now than it ever has been before. conciliatory C. 36. and the thing is easily done. "as plain as the road to the mill. that seems to be a very simple problem. many people think they understand economy when they really do not. is to be the most miserable of men. The road to wealth is. Many of my readers may say. that any person of either sex who is willing. as they do in regard to any other object which they wish to accomplish." It consists simply in expending less than we earn. Read the following passage and answer the seven questions that follow. and we know economy is wealth. In the United States. Spending money is the greatest temptation in the world. I have no doubt many of my hearers will agree it is the most difficult thing in the world to keep it. have only to set their minds upon it. exculpatory . C. and spend but nineteen pounds and sixpence is to be the happiest of mortals. where we have more land than people. There is no way to predict changes in the economy. and adopt the proper means. Dr. as Dr. to engage in any respectable occupation that offers. incredulous E. and spend twenty pounds and sixpence. at least for the time being. E. D. E. 35. one of those happy creations of the genial Dickens. Micawber. to have an income of only twenty pounds. ingenuous D." Yet I beg to say that perhaps more cases of failure arise from mistakes on this point than almost any other. we know we can't eat our cake and keep it also. it is not at all difficult for persons in good health to make money. Jacobson is about to retire. In this comparatively new field there are so many avenues of success open. may find lucrative employment. so many vocations which are not crowded. But however easy it may be found to make money. puts the case in a strong light when he says that to have annual income of twenty pounds per annum. Those who really desire to attain an independence. transportation department. whereas. The fact is. self-help manual B. getting a job B. What is the best definition of economy as it is used in this passage? A. goods. A. managing money 38.37. supportive B. patronizing E. According to the author. luxurious accommodations 40. epistle D. brochure . delegation of household affairs C. and services B. exchange of money. This passage is most likely taken from a(n) . incriminating C. novel E. excessive D. teachers D. traveling to a mill C. Who is the most likely audience for this passage? A. children 39. efficient money management D. Which word best describes the author’s attitude towards those who believe they understand money? A. less expensive E. philanthropists E. understanding the economy E. general readers C. autobiography C. incendiary 41. economists B. reading Dickens D. what is more difficult than making money? A. 8. so Dave can deliver 240 newspapers every hour. 2 3 16. Every possible combination of scores is a multiple of 7. divide 162 by 3. Prime numbers are those that are only evenly divisible by one and themselves. 11. B. In 2 hours. simply subtract the exponent in the denominator from the exponent in the numerator. perform FOIL on each answer choice until you derive the original expression. C. 7. That is. C. it would take 2 people 5 days. Resolve the parenthetical operations first. 15. E. D. The two numbers that average 2 will add up to 4. A. A set of six numbers with an average of 4 must have a collective sum of 24. D. E. 5. then multiply by 12/5. The average of these four numbers can be calculated: 20/4 = 5. 6. E. A. 2. 9. and one day of work for two 3 people would complete 1/5 of the job. 46 = 212. Simple Multiplication. B. 3. Simply substitute the given values for a and b and perform the required operations. C. A. 10. he can deliver 480 papers. To determine the number of 3-inche segments. Thus. Then multiply 24 by 0. If it takes 3 people 3 1/3 days to do the job. then it would take one person 10 1 days: 3 3 10 . 14. so the remaining numbers must add up to 20. then. When dividing exponents with the same base. There are 12 inches in a foot and 3 feet in a yard. This problem is solved by finding x in this equation: 34/80 = x/100. The two statements establish only that Alice is paid more than both Jonathan and Deborah. A. Multiply 30 by 0. This problem can be solved with the following equation. E.2 and add the product to the sale price to find the final price. in which x = the total 1 1 capacity of the tank: x x 3. they do not indicate which of these latter two is paid more. work backwards. ANSWER KEY Problem-solving 1. The resulting fraction. Substitute the given values and solve. There are 60 minutes in an hour. 17. C. must have both numerator and denominator multiplied by 4 to become a percentage. Since the cubes of a and b will retain the original sign (whether positive . 12. To solve this problem.2 and subtract this from the original price of the shirt to find the sale price: $24. Convert 20% to the fraction 1/5. 13. Four and a half yards is equal to 162 inches. Since 4 is the same as 22. 4. since the two terms of the ratio have a sum of seven. 12/25. C. 29. it will be possible to assess the relative sizes of a and b. some. 37. Here. There is no way of determining whether any. 26. A. This evidence would support the assertions of the given argument. The author seems to believe that there are plenty of lucrative jobs for everyone. C. 34. It seems clear that the author is about to describe the correct means of personal economy. 33. or none of A are C. B. it will be possible to calculate the average score. The author suggests that many people who believe they understand economy in fact do not. in which the character of the opponent rather than the merits of his reasoning is attacked. B. B. It would be possible for x and y to be negative numbers and still satisfy the conditions of (1). The author insists that many people who have no trouble earning money waste it through lavish spending. 18. A. 23. B. indicating that b is a positive number. y must be either positive or zero. 32. The author asserts both that earning money is increasingly easy and that managing money is difficult. If the two shorter sides add up to 4 inches in length. Drawing an analogy between being a member of Congress and serving on the school board is highly dubious. 30. The first statement establishes that the larger group constitutes half of the total amount of marbles. The logic of this argument can be expressed as follows: All A are B. For (2). There is no way to be certain that the Tigers lost. the author is speaking of money management on a personal or household level. 41. A. 25. For (1). 22. E. Therefore. but it then would be impossible to satisfy (2). The mayor is essentially using an ad hominem argument. As long as the sum of all five test scores can be calculated. 4. E. C. 28. 40. E. 31. D. 36. D. in which the proof depends on assumptions which themselves have not been proven. 38.or negative). The first statement only establishes that a is a positive number. C. C. meaning that the perimeter will be greater than 8 inches. according to the terms of (2). D. D. though it seems likely. B. 35. the fact that a positive number multiplied by b has a positive product establishes that b is a positive number. the two longer sides must be greater than 4 inches. This is an example of circular reasoning. C. It seems likely that this referendum could influence many young people to vote. therefore C is B. y must be odd. E. 21. 19. it does not require that a is an integer. In order for (1) to be true. C is A. which means it must be equal to 25 marbles. The negative reviews led to the poor quality of the second movie. 39. 27. and 5. A. A. 20. . 60 is the only integer between 40 and 70 that is a multiple of 2. 24. D. any positive number with a negative sign placed in front of it will become negative. This passage is clearly intended for a non-expert adult readership. SAMPLE QUESTIONS . Answer all questions following the passage on the basis of what is stated or implied in the passage. Reading Comprehension Question 1 A meteor stream is composed of dust particles that have been ejected from a parent comet at a variety of velocities. Conventional theories.modeling experiment tested this hypothesis by tracking the influence of planetary gravitation over a projected 5. Whenever the Earth passes through a meteor stream.Reading Comprehension INSTRUCTIONS The questions in this group are based on the content of a passage. Two brief periods of peak meteor activity during the shower would be observed. on average.000 years old. Astronomers have hypothesized that a meteor stream should broaden with time as the dust particles' individual orbits are perturbed by planetary gravitational fields. one as the Earth entered the thick-walled "pipe" and one as it exited. just over a day to cross the hollow. These particles follow the same orbit as the parent comet. the Earth would take. Surprisingly.000 miles per day around its orbit. Has the predicted twin-peaked activity been observed for the actual yearly Geminid meteor shower? The Geminid data between 1970 and 1979 show just such a bifurcation. Moving at a little over 1. The primary focus of the passage is on which of the following? Comparing two scientific theories and contrasting the predictions that each . the computer-model meteor stream gradually came to resemble a thick-walled.500. that the computer-model stream broadened with time. so the time interval between the two bursts of activity would vary from one year to the next. The researcher found. There is no reason why the Earth should always pass through the stream's exact center.200. a meteor shower occurs. but due to their differing velocities they slowly gain on or fall behind the disintegrating comet until a shroud of dust surrounds the entire cometary orbit.000 miles) after the first burst. After reading the passage. computer-model Geminid stream if the stream were 5. hollow pipe. the particles were randomly distributed throughout a computer simulation of the orbit of an actual meteor stream. choose the best answer to each question. In the model. however predicted that the distribution of particles would be increasingly dense toward the center of a meteor stream. as expected. The time intervals between the bursts suggest the actual Geminid stream is about 3. A recent computer. a secondary burst of meteor activity being clearly visible at an average of 19 hours (1. the Geminid.000 years old.000-year period on the positions of a group of hypothetical dust particles. The best answer is the second choice. And. The model yielded a prediction that was subsequently confirmed by observational data. the passage makes no reference to further areas for research. is consistent with the new theoretical model. and although the model yielded an unexpected result. the model was not constructed to explain the data. the last choice is not correct because it reverses the order of events. The author describes the new theoretical model in the first paragraph. the computer model confirmed the astronomers' hypothesis that meteor streams broaden with time.would make concerning a natural phenomenon Describing a new theoretical model and noting that it explains the nature of observations made of a particular natural phenomenon Evaluating the results of a particular scientific experiment and suggesting further areas for research Explaining how two different natural phenomena are related and demonstrating a way to measure them Analyzing recent data derived from observations of an actual phenomenon and constructing a model to explain the data EXPLANATION This question asks you to identify the primary focus of the passage. in the final paragraph the author states that the data obtained from actual observations which are discussed in the second and third paragraphs. The first choice is not correct. and only a single phenomenon is described in the passage. . Despite Beta's substantial technological head start and the fact that VHS was neither technically better nor cheaper than Beta. Beta producers were reluctant to form such alliances and eventually lost ground to VHS in the competition for the global VCR market. were first to develop the VCR commercially in 1975. Strategic alignments with producers of prerecorded tapes reinforced the VHS advantage. Producers of the Beta format for videocassette recorders (VCR's). Today. VCR's in the VHS format were less expensive than competing- format VCR'S. the largest payoffs may go to companies that lead in developing integrated approaches for successful mass production and distribution. The perception among consumers that prerecorded tapes were more available in VHS format further expanded VHS's share of the market. By the end of the 1980's. Beta was no longer in production According to the passage. . VHS prerecorded videotapes were more available than Beta-format tapes. consumers began to develop a preference for VCR's in the VHS format because they believed which one of the following? VCR's in the VHS format were technically better than competing-format VCR'S. for example.Reading Comprehension Question 2 Traditionally. Nor does the passage indicate that consumers believed that VHS-format VCR's were the first on the market (the third choice) or that VHS-format VCR's would eventually drive Beta VCR's out of production entirely (the last choice). The passage states that the "perception among consumers that prerecorded tapes were more available in VHS format further expanded VHS's share of the market. VCR's in the Beta format would soon cease to be produced. Seeking to maintain exclusive control over VCR distribution. EXPLANATION The best answer is the fourth choice. but producers of the rival VHS (Video Home System) format proved to be more successful at forming strategic alliances with other producers and distributors to manufacture and market their VCR format. the first firm to commercialize a new technology has benefited from the unique opportunity to shape product definitions. however. VHS was the first standard format for VCR'S. developers of VHS quickly turned a slight early lead in sales into a dominant position." None of the information given in the passage suggests that consumers thought the VHS-format was technically better (the first choice) or less expansive than Beta (the second choice). forcing followers to adapt to a standard or invest in an unproven alternative. stems. Oligosaccharins are fragments of the cell wall released by enzymes: different enzymes release different oligosaccharins. The five have so many simultaneous effects that they are not very useful in artificially controlling the growth of crops. stimulates the rate of cell elongation. each has more than one effect on the growth and development of plants.The pleiotropy of the five well-studied plant hormones is somewhat analogous to that of certain hormones in animals. ethylene. . another the ovarian follicle cells. There are indications that pleiotropic plant hormones may actually function by activating the enzymes that release these other. the five well-known plant hormones are pleiotropic rather than specific. or turned on. and inhibits the growth of lateral shoots. causes shoots to grow up and roots to grow down. and fruits? The answer is that only a small subset of the genes in a particular kind of cell are expressed. and so forth. for instance. leaves. there is a hierarchy of hormones. Auxin also causes the plant to develop a vascular system. In other words. How then can these cells differentiate and form structures as different as roots. For example. one of which stimulates the release of hormones from the adrenal cortex. Unlike the oligosaccharins. more specific chemical messengers from the cell wall. that is. Five major hormones have been identified: auxin. hormones from the hypothalamus in the brain stimulate the anterior lobe of the pituitary gland to synthesize and release many different hormones.Reading Comprehension Question 3 All of the cells in a particular plant start out with the same complement of genes. Such a hierarchy may also exist in plants. at a given time. This is accomplished by a complex system of chemical messengers that in plants include hormones and other regulatory molecules. Auxin. Studies of plants have now identified a new class of regulatory molecules called oligosaccharins. for example. and gibberellin. abscisic acid. These hormones have specific effects on target organs all over the body. and to produce ethylene. The passage suggests that which of the following is a function likely to be performed by an oligosaccharin? To stimulate a particular plant cell to become part of a plant's root system To stimulate the walls of a particular cell to produce other oligosaccharins To activate enzymes that release specific chemical messengers from plant cell walls To duplicate the gene complement in a particular plant cell To produce multiple effects on a particular subsystem of plant cells. to form lateral roots. cytokinin. One hormone stimulates the thyroid gland. they are likely to have particular effects on particular plant cells. The last paragraph characterizes oligosaccharisn as "specific chemical messengers". The last choice is incorrect because the second paragraph indicates that an oligosaccharin has a specific effect rather than multiple effects on plant cells. The fourth choice is incorrect because although oligosaccharins do affect the activity of the gene complement of a particular cell.EXPLANATION The best choice is the first. The first choice is correct because it is the only choice that describes an effect on a specific aspect of plant growth and development: stimulating a particular plant cell to become a part of a plant's root system. they do not duplicate that complement. The second and third choices are incorrect because the last paragraph indicates that enzymes activate the release of oligosaccharins. . unlike the pleiotropic hormones. The passage indicates that these chemical messengers are "specific" in that. it is unlikely that increased productivity would lead to short-term increases in stock values. As a result. major investors could cut their losses only by helping to restore their companies' productivity. . they could not sell out for a quick profit and instead had to concentrate on improving the long-term productivity of their companies. The legal limits that now prevent financial institutions from acquiring a dominant shareholding position in a corporation should be removed. today's short-term traders must be remade into tomorrow's long-term capitalistic investors. Institutions that intend to sell a large block of stock in a single corporation must give at least twenty-four hours notice of the sale. with few exceptions. the value of the stock would plummet and. The way corporations are currently run. and such institutions encouraged to take a more active role in the operations of the companies in which they invest. but on increasing the productivity of the companies in which they invest. Today. for example-and because these institutions are prohibited by antitrust laws from owning a majority of a company's stock and from actively influencing a company's decision-making. any institution that holds twenty percent or more of a company's stock should be forced to give the public one day's notice of the intent to sell those shares. United States productivity is unlikely to improve unless shareholders and the managers of the companies in which they invest are encouraged to enhance long-term productivity (and hence long-term profitability). A minority shareholder is necessarily a short-term trader.Reading Comprehension Question 4 Most large corporations in the United States were once run by individual capitalists who owned enough stock to dominate the board of directors and dictate company policy. rather than simply to maximize short-term profits. In addition. Unless the announced sale could be explained to the public on grounds other than anticipated future losses. The passage supports which of the following statements? Antitrust laws prevent any single shareholder from acquiring a majority of the stock in a corporation. the stock of large United States corporations is held by large institutions-pension funds. Such measures would force financial institutions to become capitalists whose success depends not on trading shares at the propitious moment. like the old-time capitalists. Since the return of the old-style capitalist is unlikely. Because putting such large amounts of stock on the market would only depress its value. The sudden sale of a large amount of stock in any one corporation makes the value of the stock go down. In most corporations it is the board of directors rather than the corporate managers who make policy decisions. they can enhance their wealth only by buying and selling stock in anticipation of fluctuations in its value. the author does not say whether or not other parties are similarly prohibited. directors currently play in making policy it is unlikely that increased productivity decisions. be explained to the public on grounds other than anticipated future losses. The only managers who make policy decisions. intend to sell a large block of a corporation's stock should be required to give a day's notice: the author does not say that such institutions are already required In most corporations it is the board of to do so directors rather than the corporate The third choice is not correct. The must give at least twenty-four hours author proposes that institutions that notice of the sale.causes the value shares. in the first sentence of the passage. The last choice is not the correct answer. the value of the stock would plummet… Antitrust laws prevent any single The first choice is incorrect. any institution that holds The passage states that the sudden sale of twenty percent or more of a company's a large amount of a corporation's stock stock should be forced to give the public one day's notice of the intent to sell those twenty percent or more . refers to a historical past in which individual capitalists could control boards of directors and dictate company policy. is does not say anything about the relationship between increased productivity and short-term stock values. would lead to short-term increases in stock values. The passage only discusses the relationship between increased long- term productivity and long-term profitability. . states that "large institutions" are prohibited from owning a majority of the stock in any one corporation. the passage does not say anything about what role boards of The way corporations are currently run. mention of boards of directors. The author shareholder from acquiring a majority of the stock in a corporation.EXPLANATION This question asks you to identify a statement for which evidence is presented in the passage. …In addition. The best answer is shown. However. Institutions that intend to sell a large block of stock in a single corporation The second choice is not correct. However. Unless the announced sale could of that stock to decline. the simple claim that an industry has been injured by imports is sufficient grounds to seek relief. The "United States" company claiming injury was a subsidiary of a Dutch conglomerate.Reading Comprehension Question 4 Many United States companies have." Even when no unfair practices are alleged. this quest for import relief has hurt more companies than it has helped. If competitor can prove injury from the imports-and that the United States company received a subsidy from a foreign government to build its plant abroad. while the "Canadian" companies included a subsidiary of a Chicago firm that was the second-largest domestic producer of rock salt. Perhaps the most brazen case occurred when the ITC investigated allegations that Canadian companies injuring the United States salt industry by dumping rock salt.United States company's products will be uncompetitive in the United States. The complexity of these relationships makes it unlikely that a system of import relief laws will meet the strategic needs of all the units under the same parent company. they develop an intricate web of marketing. . and research relationships. Contrary to the general impression. used to de-ice roads. unfortunately. Internationalization increases the danger that foreign companies will use import relief laws against the very companies the laws were designed to protect. The bizarre aspect of the complaint was that a foreign conglomerate with United States operations was crying for help against a United States company with foreign operations. since they would be subject to duties. Another 340 charge that foreign companies "dumped" their products in the United States at "less than fair value. made the search for legal protection from import competition into a major line of work. As corporations begin to function globally. production. Since 1980 the United States International Trade Commission (ITC) has received about 280 complaints alleging damage from imports that benefit from subsidies by foreign governments. Suppose a United States-owned company establishes an overseas plant to manufacture a product while its competitor makes the same product in the United States. owned but operate internationally may not be eligible for protection from import competition under the laws of the countries in which their plants operate. Companies in the United States that import raw materials may have to pay duties on those materials. Companies that seek legal protection from import competition may incur legal costs that far exceed any possible gain. Internationalization increases the danger The "danger" of import relief laws is that foreign companies will use import stated. The passage does not mention the situations described in the other choices. EXPLANATION This is the best answer. Companies that are United States . .The passage warns of which of the following dangers? Companies in the United States may receive no protection from imports unless they actively seek protection from import competition. Import relief laws are the legal relief laws against the very companies the laws were designed to protect. Companies that are not United States-owned may seek legal protection from import competition under United States import relief laws. protection referred to in the correct choice. If an . The length of the fencing needed is 34+20+34=88 feet. Straight lines may sometimes appear jagged. The area of the yard is 20w= 680 square feet. Problem Solving Question 1 Harriet wants to put up fencing around three sides of her rectangular yard and leave a side of 20 feet unfenced. Figures: A figure accompanying a problem solving a question is intended to provide information useful in solving the problem. Problem Solving Question 2 A toy store regularly sells all stock at a discount of 20 percent to 40 percent. Numbers: All numbers used are real numbers. so w = 680/ 20 = 34 feet. Problem Solving INSTRUCTIONS Solve the problem and indicate the best of the answer choices given. 20 feet. If the yard has an area of 680 square feet. Figures are drawn as accurately as possible EXCEPT when it is stated in a specific problem that its figure is not drawn to scale. All figures lie in a plane unless otherwise indicated. w feet. and the unknown dimension. how many feet of fencing does she need? 34 40 68 88 102 EXPLANATION W 20 20 W The diagram shows the rectangular yard with the known dimension. 60)($16) = $7.additional 25 percent were deducted from the discount price during a special sale.60) = $7.20 EXPLANATION The lowest possible price is paid when the maximum discount is received. what would be the lowest possible price of a toy costing $16 before any discount? $5.0. the lowest possible price to be paid for the item can be calculated by realizing that if you are being given a discount of 40 percent you are paying 60 percent of the listed price of the item.40($16) = $9.60 $15. Alternatively.75)(0.0. so the lowest possible regular price is $16 . . With an additional 25 percent discount.80 $9.20.20 $8. the price of the item becomes (0.60 . The best answer is the second one.25($9.20.60 $7.60. the lowest possible price is $9. If an additional 25 percent discount is offered on the item. Problem Solving Question 3 Which of the following equations has a root in common with x2 . If 1 percent equals 100 basis points. Thus the best answer is 2x2 .2 = 0. Problem Solving Question 4 If "basis points" are defined so that 1 percent is equal to 100 basis points.3 = 0 EXPLANATION Since x2 . 2(1)2 .10x .2 = 0 x2 .5 percent? .6x + 5 = (x .x .000 EXPLANATION There is a difference of 20 percent between 82. .6x + 5 = 0? x2 + 1 = 0 x2 .1). the roots of x2 . then 20 percent equals 20(100) or 2.2 = 0. only in the fourth choice does a value satisfy the equation.2 20 200 2.5)(x .5 percent.2x .5 percent is how many basis points greater than 62.5 percent and 62. When these two values are substituted in each of the five choices to determine whether or not they satisfy the equation. then 82.000 basis points.5 = 0 2x2 .2 = 0 x2 .02 .6x + 5 = 0 are 1 and 5. namely. but will not necessarily conform to the . Figures: A figure accompanying a data sufficiency problem will conform to the information given in the question. Numbers: All numbers used are real numbers.  statements (1) and (2) TOGETHER are NOT sufficient to answer the question asked. you must indicate whether  statement (1) ALONE is sufficient.Problem Solving Question 5 If x + 5y = 16 and x = -3y. but statement (1) alone is not sufficient to answer the question asked.Data Sufficiency INSTRUCTIONS This data sufficiency problem consists of a question and two statements labeled (1) and (2).  statement (2) ALONE is sufficient.  EACH statement ALONE is sufficient to answer the question asked. but NEITHER statement ALONE is sufficient. Using the data given in the statements plus your knowledge of mathematics and everyday facts (such as the number of days in July or the meaning of counterclockwise). You have to decide whether the data given in the statements are sufficient for answering the question. and additional data specific to the problem are needed. but statement (2) alone is not sufficient to answer the question asked. in which certain data are given. then y = -24 -8 -2 2 8 EXPLANATION Substitution of the second equation into the first equation yields (-3y) + 5y = 16 2y = 16 y=8 Problem Solving .  BOTH statements (1) and (2) TOGETHER are sufficient to answer the question asked. x + z = 140. angles. . According to statement (2). therefore. etc. what is the value of x ? (1) PQ = PR (2) y = 40 Explanation: According to statement (1). regions.. since x + 2y = 180 and the value of y is given. Since x + y + z = 180. (PQR is isosceles and y = z. Since statement (2) does not give a value for z. you can find the value of x. Example In triangle PQR. All figures lie in a plane unless otherwise indicated. but NEITHER statement ALONE is sufficient. y = 40. you cannot answer the question using statement (1) alone. Note: In data sufficiency problems that ask for the value of a quantity.additional information given in statements (1) and (2). Therefore. Using both statements together. exist in the order shown and that angle measures are greater than zero. you cannot answer the question using statement (2) alone. BOTH statements (1) and (2) TOGETHER are sufficient to answer the question. PQ=PR. Lines shown as straight can be assumed to be straight and lines that appear jagged can also be assumed to be straight. the data given in the statements are sufficient only when it is possible to determine exactly one numerical value for the quantity. Since statement (1) does not give a value for y. it follows that x + 2y = 180. You may assume that the positions of points. therefore. The best answer is the fourth choice. but statement (1) alone is not sufficient. (2) alone is sufficient. and thus the selling price.94.600 is 94% (100% 6%) of the selling price.600 / 0. but statement (2) alone is not sufficient. Thus.000).5($36. can be determined. EXPLANATION From (1) it follows that $84.000.600. Therefore (1) alone is sufficient. (2) The selling price was 250 percent of the original purchase price of $36. Statements (1) and (2) TOGETHER are NOT sufficient. From (2) it follows that the selling price is 2. what was the selling price of the house? (1) The selling price minus the real estate agent's commission was $84. Statement (2) ALONE is sufficient. . EACH statement ALONE is sufficient. Statement (1) ALONE is sufficient.Data Sufficiency Question 1 If a real estate agent received a commission of 6 percent of the selling price of a certain house. but NEITHER statement ALONE is sufficient. BOTH statements TOGETHER are sufficient. $84. it cannot be determined which person received the greater dollar increase. EXPLANATION In (1) there is no information about David's salary and in (2) there is no information about John's salary. (1) and (2) together are not sufficient. Statement (2) ALONE is sufficient. EACH statement ALONE is sufficient. BOTH statements TOGETHER are sufficient. Since (1) and (2) together give only the percentage increases in salary. but statement (1) alone is not sufficient. Which one received the greater dollar increase? (1) John's salary increased 8 percent. but NEITHER statement ALONE is sufficient. (2) David's salary increased 5 percent. . thus neither statement alone is sufficient. and the best answer is the last choice. Statements (1) and (2) TOGETHER are NOT sufficient. Therefore.Data Sufficiency Question 2 John and David each received a salary increase. but statement (2) alone is not sufficient. and the answer must the third or fifth choice. Statement (1) ALONE is sufficient. Statement (2) ALONE is sufficient. . Statement (2) alone is therefore not sufficient. (2) When she drives both ways. what is the distance of the shorter route? (1) When she drives from her home to her office by the shorter route and returns by the longer route. by the longer route. Statement (1) ALONE is sufficient. EXPLANATION Statement (1) alone is not sufficient because only the sum of the distances of the two routes is given and there are infinitely many pairs of numbers with a given sum. from her home to her office and back. but statement (2) alone is not sufficient. but statement (1) alone is not sufficient. she drives a total of 42 kilometers. and the third choice is the best. From (1) and (2) together. If she must also return by one of these routes. but there is no information about the distance of the shorter route. the distance of the shorter route can be determined (42 46/2).Data Sufficiency Question 3 Carlotta can drive from her home to her office by one of two possible routes. Statements (1) and (2) TOGETHER are NOT sufficient. EACH statement ALONE is sufficient. BOTH statements TOGETHER are sufficient. but NEITHER statement ALONE is sufficient. From (2) the distance of the longer route can be found. she drives a total of 46 kilometers. it follows from (2) that could represent either addition or subtraction. multiplication. What is the value of 3 2? (1) 0 1=1 (2) 1 0=1 Statement (1) ALONE is sufficient. Since 1 + 0 = 1. and 1 0 is undefined. Statement (2) ALONE is sufficient. it follows from (1) that represents addition. (2) alone is not sufficient. 0 1 = 0. but statement (1) alone is not sufficient. and 0 1 = 0. . EXPLANATION Since 0 + 1 = 1. or division. Hence. BOTH statements TOGETHER are sufficient. EACH statement ALONE is sufficient. Thus.0 = 1. but NEITHER statement ALONE is sufficient.Data Sufficiency Question 4 The symbol represents one of the following operations: addition. 0 . so that 3 2 could equal 5 or 1. 1 0 = 0.1 = -1. 1 . but statement (2) alone is not sufficient. Statements (1) and (2) TOGETHER are NOT sufficient. subtraction. so the value of 3 2 can be determined. The best answer is the first choice. (1) alone is sufficient. (2) p2 is greater than 200. then 102 + 1 = 101. and the last choice is the best answer. but statement (2) alone is not sufficient. then 62 + 1 = 37. and 17. if n = 6. Statement (1) ALONE is sufficient. For example if n = 4. BOTH statements TOGETHER are sufficient. but NEITHER statement ALONE is sufficient. EACH statement ALONE is sufficient. 37 and 101 are all prime numbers. so that p might or might not equal 37. Thus (2) alone is not sufficient. Statements (1) and (2) TOGETHER are NOT sufficient. . where n is an integer. Statement (2) ALONE is sufficient. Since 142 = 196 and 152 = 225. EXPLANATION In (1) the expression n2 + 1 can represent a prime number less than 37. depending on the value of n. Thus (1) alone is not sufficient.Data Sufficiency Question 5 Is the prime number p equal to 37? (1) p = n2 + 1. but statement (1) alone is not sufficient. and if n = 10. then 42 + 1 = 17. it follows from (2) that p > 14. The values of p for n = 4 and n = 6 given above show that (1) and (2) together are not sufficient. or greater than 37. equal to 37.
Copyright © 2024 DOKUMEN.SITE Inc.